Linear Transformation and Magnitudes

Click For Summary
SUMMARY

The discussion focuses on proving that for a linear transformation T: Rm → Rn, there exists a constant M such that |T(h)| ≤ M|h| for all h in Rm. The key equations defining T as a linear transformation are T(x+y) = T(x) + T(y) and T(cx) = cT(x). The solution approach involves expressing |T(h)| and |h| in terms of their matrix representation A, leading to the conclusion that the function f: Rm → R defined by f(x) = |T(x)| achieves a maximum on the unit sphere Sm-1.

PREREQUISITES
  • Understanding of linear transformations in vector spaces
  • Familiarity with matrix representations of linear transformations
  • Knowledge of norms and magnitudes in Rm
  • Basic concepts of calculus on manifolds
NEXT STEPS
  • Study the properties of linear transformations in depth
  • Learn about matrix norms and their implications on linear mappings
  • Explore the concept of unit spheres in Rm and their significance in optimization
  • Investigate the relationship between linear transformations and eigenvalues
USEFUL FOR

Students of mathematics, particularly those studying linear algebra and calculus on manifolds, as well as educators and researchers interested in the properties of linear transformations and their applications.

Nebula
Messages
45
Reaction score
0

Homework Statement


From Calculus on Manifolds by Spivak: 1-10
If T:Rm -> Rn is a Linear Transformation show that there is a number M such that |T(h)| \leq M|h| for h\inRm

Homework Equations


T is a Linear Transformation
=> For All x,y \in Rn and scalar c
1. T(x+y)=T(x)+T(y)
2. T(cx)=cT(x)

The Attempt at a Solution


Well I didn't get very far but I do know this. The matrix of T with respect to the standard basis is A. Ah=T(h)
So we can write what |T(h)| and |h| look like.
|T(h)|=sqrt((a11h1+...+a1nhn)2+...+(am1h1+...+amnhn)2)
where aij's are the elements in the matrix representation of T, A.

|h|=sqrt((h1)2+...+(hn)2)

:confused: Suggestions, hints and clues are all appreciated! :smile:
 
Physics news on Phys.org
Try to show that the function f:\mathbb{R}^m\to\mathbb{R}, f(x)=|T(x)| has a maximum when restricted to the unit sphere S^{m-1} in \mathbb{R}^m. Then use the fact that every nonzero vector can be written in the form \lambda v, where \lambda is a scalar and v is a unit vector.
 
Question: A clock's minute hand has length 4 and its hour hand has length 3. What is the distance between the tips at the moment when it is increasing most rapidly?(Putnam Exam Question) Answer: Making assumption that both the hands moves at constant angular velocities, the answer is ## \sqrt{7} .## But don't you think this assumption is somewhat doubtful and wrong?

Similar threads

  • · Replies 3 ·
Replies
3
Views
2K
  • · Replies 11 ·
Replies
11
Views
2K
Replies
2
Views
1K
  • · Replies 43 ·
2
Replies
43
Views
4K
  • · Replies 2 ·
Replies
2
Views
1K
Replies
4
Views
2K
  • · Replies 4 ·
Replies
4
Views
2K
  • · Replies 3 ·
Replies
3
Views
2K
  • · Replies 1 ·
Replies
1
Views
2K
  • · Replies 2 ·
Replies
2
Views
1K